Putnam 2017 풀이 - 풀리는 것만

2018. 11. 30. 10:12수학 문풀/기타

AoPS 링크


A1. 


다음을 만족하는 최소의 집합 \(S\)에 속하지 않는 원소로는 어떤 것이 있겠는가?

- \(2 \in S\).

- \(n^2 \in S \implies n \in S\).

- \(n \in S \implies (n+5)^2 \in S\).



A2.

다음의 점화식을 만족하는 함수열 \(Q_{n}\)이 모두 정수계수 다항식임을 보여라.

- \(Q_{0}(x) = 1, Q_{1}(x) = x\)

- \(Q_{n}(x) Q_{n-2}(x) = \{ Q_{n-1} (x) \}^{2} - 1\)



A3.


함숫값이 양수인 연속함수 \(f,g\)를 생각하자. \(\int_{a}^{b} f = \int_{a}^{b} g \)이지만 \(f \not\equiv g\)이다. 이 때 다음의 수열 \(\{I_{n}\}\)을 생각하자.


$$ I_{n} := \int_{a}^{b} \frac{f^{n+1}}{g^{n}} dx $$


\(I_{n}\)이 증가수열이고, \(\lim_{n\to\infty}I_{n} = \infty\)임을 보여라.



A5.


NacChal에 나왔다.




B3.


0 또는 1의 값을 갖는 수열 \(c_{i}\)에 대해, \(f(x) = \sum_{i=0}^{\infty} c_{i}x^{i}\)를 정의하자.

\(f(\frac{2}{3}) = \frac{3}{2}\)일 때, \(f(\frac{1}{2})\)는 무리수임을 보여라.



'수학 문풀 > 기타' 카테고리의 다른 글

KAIST POW 2019-02 Simplification of an expression with factorials  (0) 2019.03.29
Balkan MO 2015  (0) 2019.02.09
2018 대수경 2분야 5번 풀이  (0) 2018.11.24
2017 Benelux MO 풀이  (0) 2018.09.29
IMO Shortlist 2000  (0) 2017.12.31